Đến nội dung

Hình ảnh

Một số bài toàn bất đẳng thức mình cần được giúp đỡ!

bất đẳng thức bunhia

  • Please log in to reply
Chủ đề này có 19 trả lời

#1
HocSinhGioiii

HocSinhGioiii

    Lính mới

  • Thành viên mới
  • 5 Bài viết

Mọi người xuất thủ tương trợ em với ạ :V
66963062_470616080390789_504091774854522
66831715_915422062140901_436699461910842
66427953_338074600450417_218376038059606



#2
Gammaths11

Gammaths11

    Hạ sĩ

  • Thành viên
  • 53 Bài viết

bài 15: đặt vế trái là A 

 xét biểu thức B=$a(a^{2}+8bc)+b(b^{2}+8ac)+c(c^2+8ab)$

Áp dụng bất đẳng thức Holder với 3 dãy ta có:

A.A.B$\geq (a+b+c)^{3}$

ta cần chứng minh $(a+b+c)^{3} \geq B$ 

$\Leftrightarrow (a+b+c)^{3}\geq a^{3}+b^{3}+c^{3}+24abc \Leftrightarrow c(a-b)^{2}+a(b-c)^{2}+b(c-a)^{2}\geq 0$



#3
Sin99

Sin99

    Thượng sĩ

  • Thành viên
  • 238 Bài viết

Bài 17. $ VP = \sum \sqrt{x-1} = \sum \sqrt{x(1-\frac{1}{2}} \leq \sqrt{(x+y+z)(3-\sum \frac{1}{x})} = \sqrt{(x+y+z)(3-2)} = VT $



#4
Sin99

Sin99

    Thượng sĩ

  • Thành viên
  • 238 Bài viết

Bài 19. Bằng U.C.T ta cần chứng minh $ \frac{1}{2-a} \geq \frac{a^2}{2} + \frac{1}{2} \Leftrightarrow \frac{a(a-1)^2}{2-a} \geq 0 $ (Đúng). Tương tự, cộng theo vế ta có ĐPCM



#5
Gammaths11

Gammaths11

    Hạ sĩ

  • Thành viên
  • 53 Bài viết

Bài 17. $ VP = \sum \sqrt{x-1} = \sum \sqrt{x(1-\frac{1}{2}} \leq \sqrt{(x+y+z)(3-\sum \frac{1}{x})} = \sqrt{(x+y+z)(3-2)} = VT $

giải bài 16 18 vs



#6
Sin99

Sin99

    Thượng sĩ

  • Thành viên
  • 238 Bài viết

Bài 20,21 tương tự bài 19



#7
Gammaths11

Gammaths11

    Hạ sĩ

  • Thành viên
  • 53 Bài viết

Bài 20,21 tương tự bài 19

bài 20 UCT thế nào vậy



#8
toanND

toanND

    Binh nhất

  • Thành viên mới
  • 49 Bài viết

$\boxed{16}$

Áp dụng BĐT Cauchy Schwarz ta có

$\frac{a}{4b^2+1}+\frac{b}{4c^2+1}+\frac{c}{4a^2+1}=\frac{a^3}{4a^2b^2+a^2}+\frac{b^3}{4b^2c^2+c^2}+\frac{c^3}{4c^2a^2+c^2}\geq \frac{(a\sqrt{a}+b\sqrt{b}+c\sqrt{c})^2}{4(a^2b^2+b^2c^2+c^2a^2)+a^2+b^2+c^2}$

Ta chỉ cần chứng minh 

$4(a^2b^2+b^2c^2+c^2a^2)+a^2+b^2+c^2\leq 1 =(a+b+c)^2$

$\Leftrightarrow ab(1-2ab)+bc(1-2bc)+ca(1-2ca)\geq0$

Lại có $ab>0$ , $ab\leq\frac{(a+b)^2}{4}<\frac{(a+b+c)^2}{4}$ $\Rightarrow ab<\frac{1}{4}\Rightarrow ab(1-2ab)>0$

Tương tự ta có đpcm

Dấu = không xảy ra


______________ :lol:  :lol:  :lol:  :lol:  :lol:  :lol: ______________

         


#9
Gammaths11

Gammaths11

    Hạ sĩ

  • Thành viên
  • 53 Bài viết

bài 25: $\sum \frac{3\sqrt{bc}}{a+3\sqrt{bc}}\doteq 3-\sum \frac{a}{a+3\sqrt{bc}}\leq 3-\frac{(\sqrt{a}+\sqrt{b}+\sqrt{c})^{2}}{\sum a+3\sum \sqrt{bc}}\leq 3-\frac{(\sqrt{a}+\sqrt{b}+\sqrt{c})^{2}}{(\sqrt{a}+\sqrt{b}+\sqrt{c})^{2}+\frac{(\sqrt{a}+\sqrt{b}+\sqrt{c})^{2}}{3}} =3-\frac{3}{4}=\frac{9}{4}$



#10
Gammaths11

Gammaths11

    Hạ sĩ

  • Thành viên
  • 53 Bài viết

bài 23: $\sum \frac{3a+b}{2a+c}=3+\sum \frac{a+b-c}{2a+c}$

bđt $\Leftrightarrow \sum \frac{a+b-c}{2a+c}\geq 1$

ta có: $\sum \frac{a+b-c}{2a+c}= \sum \frac{(a+b-c)^{2}}{(a+b-c)(2a+c)}\geq \frac{(a+b+c)^{2}}{\sum (a+b-c)(2a+c)}\doteq \frac{(a+b+c)^{2}}{a^{2}+b^{2}+c^{2}+2ab+2bc+2ca}= 1$



#11
PhamQuangDuong

PhamQuangDuong

    Lính mới

  • Thành viên mới
  • 5 Bài viết

21, $3-P=\Sigma\frac{1}{x+1} \ge \frac{1+1+1}{x+y+z+3}=\frac{9}{4 }$

$\Rightarrow P \le \frac{3}{4 }$



#12
PhamQuangDuong

PhamQuangDuong

    Lính mới

  • Thành viên mới
  • 5 Bài viết

17, $ \Sigma \frac{1}{x} = 2 \Rightarrow \Sigma \frac{x-1}{x} = 1$

Có $x+y+z=(x+y+z).\Sigma\frac{x-1}{x}\ge(x+y+z).\frac{\Sigma\sqrt{x-1}}{x+y+z}=VP$ (Q.E.D)



#13
PhamQuangDuong

PhamQuangDuong

    Lính mới

  • Thành viên mới
  • 5 Bài viết

bài 23: $\sum \frac{3a+b}{2a+c}=3+\sum \frac{a+b-c}{2a+c}$

bđt $\Leftrightarrow \sum \frac{a+b-c}{2a+c}\geq 1$

ta có: $\sum \frac{a+b-c}{2a+c}= \sum \frac{(a+b-c)^{2}}{(a+b-c)(2a+c)}\geq \frac{(a+b+c)^{2}}{\sum (a+b-c)(2a+c)}\doteq \frac{(a+b+c)^{2}}{a^{2}+b^{2}+c^{2}+2ab+2bc+2ca}= 1$

sao lại thế này ạ ?



#14
PhamQuangDuong

PhamQuangDuong

    Lính mới

  • Thành viên mới
  • 5 Bài viết

bài 20 hình như UCT bị ngược dấu ?



#15
Sin99

Sin99

    Thượng sĩ

  • Thành viên
  • 238 Bài viết

Bài 18.(sol by Nguyễn Huy Thắng) Sử dụng phương pháp S.O.S, ta có

$BĐT  \Leftrightarrow \sum (\frac{1}{4a^2-bc+1} -\frac{1}{2}) \geq 0 \Leftrightarrow \sum \frac{(c-a)(2a+b)-(a-b)(2a+c)}{2(4a^2-bc+1)} \geq 0 \Leftrightarrow \sum (a-b)(\frac{2b+c}{4b^2-ac+1}-\frac{2a+c}{4a^2-bc+1}) \geq 0 \Leftrightarrow \sum (a-b)^2(6ab+4ac+4bc+c^2) \geq 0$ (Đúng).


Bài viết đã được chỉnh sửa nội dung bởi Sin99: 13-07-2019 - 19:04


#16
Gammaths11

Gammaths11

    Hạ sĩ

  • Thành viên
  • 53 Bài viết

sao lại thế này ạ ?

dùng cauchy-schwarz dạng engel


Bài viết đã được chỉnh sửa nội dung bởi Gammaths11: 13-07-2019 - 20:03


#17
HocSinhGioiii

HocSinhGioiii

    Lính mới

  • Thành viên mới
  • 5 Bài viết

Mọi người cố gắng thử dùng C-S  ( Bunhia) được không ạ? :V



#18
bangvoip673

bangvoip673

    Binh nhất

  • Thành viên mới
  • 37 Bài viết

ptnk ẻ có khác



#19
Gammaths11

Gammaths11

    Hạ sĩ

  • Thành viên
  • 53 Bài viết

ptnk ẻ có khác

đag đợi pbc-ẻ xuất chiêu mà chưa thấy nhác thế



#20
Gammaths11

Gammaths11

    Hạ sĩ

  • Thành viên
  • 53 Bài viết

bài 22

$\frac{1+a}{1-a}=1-\frac{2a}{1-a}$

BĐT $\Leftrightarrow 3-\sum \frac{2a}{1-a}\leq 2\left ( \frac{b}{a}+\frac{c}{b}+\frac{a}{c} \right )\Leftrightarrow \sum 2a\left ( \frac{1}{c}-\frac{1}{b+c} \right )\geq 3\Leftrightarrow \sum \frac{ab}{c(b+c)}\geq \frac{3}{2}$

ÁP dụng bunhia :$\left ( \sum \frac{ab}{c(b+c)} \right )\left ( \sum (b+c) \right )\geq \left ( \sum \sqrt{\frac{ab}{c}} \right )^{2}$

lại có: $(x+y+z)^{2}\geq 3(xy+yz+zx)$$\Rightarrow \left ( \sum \sqrt{\frac{ab}{c}} \right )^{2}\geq 3\left ( \sum \sqrt{\frac{ab}{c}.\frac{bc}{a}} \right )\doteq 3(a+b+c)$

$\Rightarrow 2(a+b+c)\left ( \sum \frac{ab}{c(b+c)} \right )\geq 3(a+b+c)\Rightarrow \sum \frac{ab}{c(b+c)}\geq \frac{3}{2}$


Bài viết đã được chỉnh sửa nội dung bởi Gammaths11: 15-07-2019 - 07:49






Được gắn nhãn với một hoặc nhiều trong số những từ khóa sau: bất đẳng thức bunhia

0 người đang xem chủ đề

0 thành viên, 0 khách, 0 thành viên ẩn danh